Prove this inequality $\sum_{r=1}^{n}a_{r}\sqrt{\frac{n-1}{1-a_{r}}}\ge\sum_{r=1}^{n}\sqrt{a_{r}}$

a.m.-g.m.-inequalitycauchy-schwarz-inequalityinequalityjensen-inequality

Let $({a_{r}})_{r=1}^{n}$ be a sequence of $n$ positive real numbers that sum to 1. Prove that for all $n>1$ :\begin{align}
\sum_{r=1}^{n}a_{r}\sqrt{\frac{n-1}{1-a_{r}}}&\ge\sum_{r=1}^{n}\sqrt{a_{r}}.
\end{align}

$(x_1, x_2, \ldots, x_n)$, and $(y_1, y_2, \ldots, y_n)$,
$$\left(\sum_{i=1}^{n} x_iy_i\right)^2 \leq \left(\sum_{i=1}^{n} x_i^2\right)\left(\sum_{i=1}^{n} y_i^2\right).$$

$$\begin{align*} x_i &= a_i\sqrt{\frac{n-1}{1-a_i}}, \ y_i &= \sqrt{a_i}. \end{align*}$$

Using cauchy-Schwarz inequality

$$\left(\sum_{i=1}^{n} a_i\sqrt{\frac{n-1}{1-a_i}}\cdot \sqrt{a_i}\right)^2 \leq \left(\sum_{i=1}^{n} \left(a_i\sqrt{\frac{n-1}{1-a_i}}\right)^2\right)\left(\sum_{i=1}^{n} \left(\sqrt{a_i}\right)^2\right).$$

$$\left(\sum_{i=1}^{n} a_i\sqrt{\frac{n-1}{1-a_i}}\cdot \sqrt{a_i}\right)^2 \leq \left(\sum_{i=1}^{n} \frac{n-1}{1-a_i}\cdot a_i\right)\left(\sum_{i=1}^{n} a_i\right).$$

$$\left(\sum_{i=1}^{n} a_i\sqrt{\frac{n-1}{1-a_i}}\cdot \sqrt{a_i}\right)^2 \leq \left(\sum_{i=1}^{n} \frac{n-1}{1-a_i}\cdot a_i\right)$$

$$\sum_{i=1}^{n} \frac{n-1}{1-a_i}\cdot a_i = (n-1)\sum_{i=1}^{n} \frac{a_i}{1-a_i}.$$

Sir Michael Rozenberg proved this with Jensen's inequality but can someone help me to prove this inequality with holder's inequality

Best Answer

Another way.

From my previous post we need to prove that: $$\sum_{r=1}^na_r\sqrt{\frac{n-1}{n-a_r}}\geq\sum_{r=1}\sqrt{a_r},$$ where $\sum\limits_{r=1}^na_r=n$ and $a_r>0$.

Now, by C-S $$n=\sqrt{n^2}=\sqrt{\sum_{r=1}^n1\sum_{r=1}^n}a_r\geq\sum_{r=1}^n\sqrt{a_r}$$ and by Holder we obtain: $$\sum_{r=1}^na_r\sqrt{\frac{n-1}{n-a_r}}=\sqrt{\frac{(n-1)\left(\sum\limits_{r=1}^n\frac{a_r}{\sqrt{n-a_r}}\right)^2\sum\limits_{r=1}^na_r(n-a_r)}{\sum\limits_{r=1}^na_r(n-r)}}\geq$$ $$\geq\sqrt{\frac{(n-1)\left(\sum\limits_{r=1}^na_r\right)^3}{\sum\limits_{r=1}^na_r(n-a_r)}}=\sqrt{\frac{(n-1)n^3}{\sum\limits_{r=1}^n(na_r-a_r^2-n+1)+n(n-1)}}=$$ $$=\sqrt{\frac{(n-1)n^3}{\sum\limits_{r=1}^n(a_r-1)(n-1-a_r)+n(n-1)}}=$$ $$=\sqrt{\frac{(n-1)n^3}{\sum\limits_{r=1}^n(a_r-1)(n-1-a_r-n+2)+n(n-1)}}=$$ $$=\sqrt{\frac{(n-1)n^3}{-\sum\limits_{r=1}^n(a_r-1)^2+n(n-1)}}\geq\sqrt{\frac{(n-1)n^3}{n(n-1)}}=n\geq\sum_{r=1}^n\sqrt{a_r}$$ and we are done.